Question and Answers Forum

All Questions      Topic List

Arithmetic Questions

Previous in All Question      Next in All Question      

Previous in Arithmetic      Next in Arithmetic      

Question Number 3929 by Filup last updated on 25/Dec/15

For:  S=1+(1/2)+(1/3)+...+(1/n)  S=H_n     Harmonic sequence  H_n =Σ_(i=1) ^n (1/i)  Can you solve the partial sum?

$$\mathrm{For}: \\ $$$${S}=\mathrm{1}+\frac{\mathrm{1}}{\mathrm{2}}+\frac{\mathrm{1}}{\mathrm{3}}+...+\frac{\mathrm{1}}{{n}} \\ $$$${S}={H}_{{n}} \:\:\:\:{Harmonic}\:{sequence} \\ $$$${H}_{{n}} =\underset{{i}=\mathrm{1}} {\overset{{n}} {\sum}}\frac{\mathrm{1}}{{i}} \\ $$$${Can}\:{you}\:{solve}\:{the}\:{partial}\:{sum}? \\ $$

Commented by Yozzii last updated on 25/Dec/15

Wolfram Alpha gives                     H_n =ψ_0 (n+1)+γ  where γ−Euler Mascheroni constant  ψ_0 (z) is the digamma function defined  by the logarithmic derivative of the  gamma function Γ(z).             ψ_0 (z)=(d/dz){lnΓ(z)}=((Γ^′ (z))/(Γ(z)))  The digamma function satisfies                 ψ_0 (z)=∫_0 ^∞ ((e^(−t) /t)−(e^(−zt) /(1−e^(−t) )))dt.  For z≡n , n∈Z^+ ,  ψ_0 (n)=−γ+Σ_(i=1) ^(n−1) (1/i)⇒H_(n−1) =ψ_0 (n)+γ.

$${Wolfram}\:{Alpha}\:{gives}\: \\ $$$$\:\:\:\:\:\:\:\:\:\:\:\:\:\:\:\:\:\:{H}_{{n}} =\psi_{\mathrm{0}} \left({n}+\mathrm{1}\right)+\gamma \\ $$$${where}\:\gamma−{Euler}\:{Mascheroni}\:{constant} \\ $$$$\psi_{\mathrm{0}} \left({z}\right)\:{is}\:{the}\:{digamma}\:{function}\:{defined} \\ $$$${by}\:{the}\:{logarithmic}\:{derivative}\:{of}\:{the} \\ $$$${gamma}\:{function}\:\Gamma\left({z}\right). \\ $$$$\:\:\:\:\:\:\:\:\:\:\:\psi_{\mathrm{0}} \left({z}\right)=\frac{{d}}{{dz}}\left\{{ln}\Gamma\left({z}\right)\right\}=\frac{\Gamma^{'} \left({z}\right)}{\Gamma\left({z}\right)} \\ $$$${The}\:{digamma}\:{function}\:{satisfies}\: \\ $$$$\:\:\:\:\:\:\:\:\:\:\:\:\:\:\psi_{\mathrm{0}} \left({z}\right)=\int_{\mathrm{0}} ^{\infty} \left(\frac{{e}^{−{t}} }{{t}}−\frac{{e}^{−{zt}} }{\mathrm{1}−{e}^{−{t}} }\right){dt}. \\ $$$${For}\:{z}\equiv{n}\:,\:{n}\in\mathbb{Z}^{+} , \\ $$$$\psi_{\mathrm{0}} \left({n}\right)=−\gamma+\underset{{i}=\mathrm{1}} {\overset{{n}−\mathrm{1}} {\sum}}\frac{\mathrm{1}}{{i}}\Rightarrow{H}_{{n}−\mathrm{1}} =\psi_{\mathrm{0}} \left({n}\right)+\gamma. \\ $$$$ \\ $$

Terms of Service

Privacy Policy

Contact: info@tinkutara.com